6. Find the value of x to the nearest tenth.

6. Find The Value Of X To The Nearest Tenth.

Answers

Answer 1

Answer:

Step-by-step explanation:

[tex]Cos \ 39 = \frac{adjacent \ side}{hypotenuse}\\\\0.7771 = \frac{7}{x}[/tex]

x * 0.7771 = 7

[tex]x =\frac{7}{0.7771}=9.007[/tex]

x = 9


Related Questions

The simple interest on certain principal is 1/5 of the amount in 5 years, find the rate of interest.
a. 5%
b. 80%
c 6 2/3
d 10%​

Answers

the answer to is percent is b

The rate is 4% per year.

It is required to find the rate of interest.

What is simple interest ?

Simple Interest (S.I.) is the method of calculating the interest amount for a particular principal amount of money at some rate of interest.

Given that:

Let, principal is P,

so, interest is P/5 in 5 yrs.

As from the formula of simple interest,

So, P/5 = P× R ×5/ 100 (R = rate)Or, R = 4%

So, rate is 4% per year.

Learn more about simple interest here:

https://brainly.com/question/24181542

#SPJ2

The missing option is e. 4%

Solve x2 + 10x = 24 by completing the square. Which is the solution set of the equation?

(negative 5 minus StartRoot 34 EndRoot comma negative 5 + Startroot 34 EndRoot)
(negative 5 minus StartRoot 29 EndRoot comma negative 5 + StartRoot 29 EndRoot)
{–12, 2}
{–2, 12}

Answers

Answer:

(-12,2)

Step-by-step explanation:

x^2 + 10x = 24

x^2 + 10x + (10/2)^2 = 24 + (10/2)^2

10/2 = 5

5^2 = 25

x^2 + 10x + 25 = 24 + 25

x^2 + 10x + 25 = 49

(x + 5)^2 = 49                 Take the square root of both sides

(x + 5) = sqrt(49)

x + 5 = +/- 7

x = +/- 7 - 5

x = +7 - 5 = 2

x = -7 - 5 = -12

Answer:

{ -12 , 2}

Step-by-step explanation:

x² + 10x = 24

In order to complete the square, the equation must first be in the form x² + bx =c.

x² + 10x = 24

Divide 10, the coefficient of the x term, by 2 to get 5. Then add the square of 5 to both sides of the equation. This step makes the left hand side of the equation a perfect square.

x² + 10x + 5² = 24 + 5²

expand exponents.

x² + 10x + 25 = 24 + 25

Add 24 and 25

x² + 10x + 25 = 49

Factor x² + 10x + 25. In general, when x² + bx + c is a perfect square, it can always be factored as ( x + b/2)².

( x + 5 )² =49

Take the square root of both sides of the equation.

[tex] \small \sf \sqrt{(x + 5) {}^{2} } = \sqrt{49} [/tex]

simplify

x + 5 = 7x + 5 = +/- 7

Subtract 5 from both sides.

x + 5 - 5 = 7 - 5

x = 2

x + 5 - 5 = +/- 7 -5

x = -7 - 5 = -12

What is the length of AC?
A
8
B
C
O

Answers

Answer:

hi, it's 16 because:

[tex]ab = bc \\ ab = 8 \\ ac = 2 \times 8 = 16 \\ ac = 16[/tex]

Answer:

16

Step-by-step explanation:

Theorem: In a circle, if a radius is perpendicular to a chord, then the radius is the perpendicular bisector of the chord.

In this case, the radius containing points O and B is perpendicular to chord AC, so the radius bisects chord AC making AB = BC.

Also,

AB + BC = AC

By substitution, we have

AB + AB = AC

AC = 2AB

AC = 2(8)

AC = 16

Annie invests $400 in a bank that offers 5.5% simple annual interest after 6 years her investment will increase by blank dollars

Answers

Answer:

Her investment will increase by 551.54 dollars

Step-by-step explanation:

So we know that the exponential function formula is f(x)=a(1+r)^x

So knowing that we can input values

So now we have f(x)=400(1+0.055)^6

Since it is annual we will only have the interest yearly so that changes are equation to look like this f(x)=400(1+0.055/1)^6

So next we have to add 1 and 0.055 which is 1.055

So we input that into the equation so now we have f(x)=400(1.055/1)^6

Now we have to do (1.055/1) to the power of six, so we get 1.37884280676

Now we have to 1.37884280676 times 400,  which would be 551.537122705.

Assuming they want to the nearest 100th it would be 551.54

So the answer is 551.54 dollars

Hopefully, that helped. If I made any mistake or I am incorrect feel free to correct me. :)

Plz help me solve this ASAP and show the work thank you

Answers

Answer:

14

Step-by-step explanation:

The diagram is right angle triangle so we can use SOHCAHTOA

So in the diagram we have 62 degree opposite to x and the hypotenuse

Step 1

Sin62=x/16

Step 2

X=16sin62 by cross multiplication

Step 3

X=14.13

X=14

help me pls I dont get this

Answers

Answer:

D

Step-by-step explanation:

The answer is D because if you flip those circles down and wrap the rectangle around it will create a cylinder

Instructions: Find AB. given that line AD is the perpendicular bisector of BC.

Answers

Answer:

19

Step-by-step explanation:

because line a d divided the triangle ABC into two equal halves therefore line a b is equal to line AC which is equal to 19

Does (2, 1) make the equation y = 8x true? yes no​

Answers

Answer:

No

Step-by-step explanation:

Because 1 don't equal to 16

Answer:

no

Step-by-step explanation:

* means multiply

(2,1)

x = 2

y = 1

just plug in the numbers

1 = 8*2 ?

1 = 16? no

whats the function for y=-4x+3

Answers

Answer:

the function for y=-4x+3

= soln,

y=mx+b"

Please help me with this problem

Answers

Answer:

Step-by-step explanation:

a*b *c = abc

15c = 15 . c

2a * 10b = 2* 10 * a * b = 20ab

The above mentioned expressions are true

That are false

12ab = 12a * 12b   ----> false because 12ab = 12 * a* b

7a *7b = 14ab is false because   7a * 7b = 7 * 7* a *b = 49ab

For each journey between work and home, Arjun uses 1.3 gallons of petrol. Arjun has 52 litres of petrol in his car. How many complete journeys between work and home can he do?
1 gallon = 4.5 litres.

Answers

Answer: 9 full journeys

Step-by-step explanation:

4.5 litres * 1.3 gallons = 5.8 litres of petrol

52 litres ÷ 5.8 litres of petrol = 8.9 journeys ≈ 9 full journeys

What is the answer to this question

Answers

your answer would be C
if triangle efg is equal to hjk then it will still be equal the opposite way. The symmetric property tells us that both sides of the equal sign are equal no matter what side they are on.

What is the slope of the line? What is the y-intercept of the line? y = 1

Answers

Answer:

m = 0

y intercept = 1

Step-by-step explanation:

The given equation of the line is ,

[tex]\implies y = 1[/tex]

We know that the Standard equation of Slope Intercept Form of the line is,

[tex]\implies y = mx + c[/tex]

Where ,

m is slope c is y intercept

On comparing to the Standard form of the line we get ,

[tex]\implies Slope = 0 [/tex]

[tex]\implies y - intercept= 1[/tex]

Slope of the line is 0, since y=mx+c, if m = 0, then y = c, where the constant (c)=1

y intercept of the line is y = 1.

Answered by GAUTHMATH

What is the other solution to the equation (Algebra ll) *URGENT*

Answers

Given:

The equation is:

[tex]3-2|0.5x+1.5|=2[/tex]

One solution of this equation is -2.

To find:

The another solution of the given equation.

Solution:

We have,

[tex]3-2|0.5x+1.5|=2[/tex]

It can be written as:

[tex]-2|0.5x+1.5|=2-3[/tex]

[tex]-2|0.5x+1.5|=-1[/tex]

Divide both sides by -2.

[tex]|0.5x+1.5|=0.5[/tex]

After removing the modulus, we get

[tex]0.5x+1.5=\pm 0.5[/tex]

Case I:

[tex]0.5x+1.5=0.5[/tex]

[tex]0.5x=0.5-1.5[/tex]

[tex]0.5x=-1[/tex]

Divide both sides by 0.5.

[tex]x=-2[/tex]

Case II:

[tex]0.5x+1.5=-0.5[/tex]

[tex]0.5x=-0.5-1.5[/tex]

[tex]0.5x=-2[/tex]

Divide both sides by 0.5.

[tex]x=-4[/tex]

One solution of the given equation is [tex]x=-2[/tex] and the another one is [tex]x=-4[/tex].

Therefore, the correct option is B.

What is the measure of side B’C’?

Answers

36 bc it has 3 different sides and it all equals

The sales tax rate for the state of Washington was 7.6%.

What is the state sales tax on a $5,300 car in WashingtoN

What is the final cost of the car, including tax?

Answers

Answer:

Sales tax: 402.80 Final cost: 5,702.80

Step-by-step explanation:

Sales price x sales tax rate = sales tax

5300 x .076 (7.6%) = 402.80

Sales price + tax = final cost

5300 + 402.80 = 5702.80

A department store holds a year-end clearance sale that includes a 5.5% discount on cosmetics. Find the sale price of a bottle of perfume if its original price was $48.41.

Answers

Answer:

Sale price = $45.75

Step-by-step explanation:

Original price = $48.41

Percentage Discount = 5.5%

Amount of discount = Percentage Discount × Original price

= 5.5% × $48.41

= 5.5/100 × $48.41

= 0.055 × $48.41

= $2.66255

Sale price = Original price - Amount of discount

= $48.41 - $2.66255

= $45.74745

Approximately,

Sale price = $45.75

What is the measure of x in the figure below?

Answers

Answer:

115 degree

Step-by-step explanation:

x + 65 = 180 (Sum of angles in a straight line = 180 [Also known as Linear pair])

x = 180 - 65

x = 115 degree

Max earned $8 for each hour of work he completed. His mom put $120 in his savings account at the end of the summer. By the end of the summer, Max had $400 in his savings account. Write an equation to determine the number of hours Max worked over the summer.

Answers

Answer:

8x +120= 400

Step-by-step explanation:

Let the number of hours Max worked over the summer be x.

Assuming that he had $0 in his bank at the start of summer,

Total savings= 8(number of hours) +120

400= 8x +120

8x +120= 400

To find the value of x, first bring all the constants to one side of the equation.

8x= 400 -120

Simplify:

8x= 280

Divide both sides by 8:

x= 280 ÷8

x= 35

∴ Max worked for 35 hours over the summer.


PLEASE HELP.
if v1….

Answers

I believe the answer is 4.4

convert 9.56 cm = ________________ M​

Answers

Answer:

[see below]

Step-by-step explanation:

There are 100 cm in a meter.

9.56/100 = 0.0956

Hope this helps you.

(PS: You can mark the other user as Branliest)

Will give brainliest!!!
2.6(5.5p – 12.4) = 127.92
p=

Answers

Answer:

[tex]p=11.2[/tex]

Step-by-step explanation:

Given [tex]2.6(5.5p-12.4)=127.92[/tex], distribute to remove the parentheses:

[tex]2.6\cdot 5.5p-2.6\cdot 12.4=127.92[/tex]

Simplify:

[tex]14.3p-32.24=127.92[/tex]

Add 32.24 to both sides:

[tex]14.3p=160.16[/tex]

Divide both sides by 14.3:

[tex]p=\frac{160.16}{14.3}=\boxed{11.2}[/tex]

given the following diagram, find the required measure.

Answers

Answer:

m<6=90

Step-by-step explanation:

the answer is 90 or D

Answer:

90

Step-by-step explanation:

If 1 = 140, then 2 = 40. 180-140=40

If 2 = 40 and 3 = 50, then 6 = 90. 180 - 40 - 50 = 90

Linear angles = 180

Triangle Sum Theorem = Sum of all angles in a triangle = 180

A marketing researcher interviews a large number of respondents and asks them a set of questions that are listed in a questionnaire. The respondents are required to select a response from a given set of options. In this case, the researcher is _____.

Answers

Answer: conducting quantitative research

Step-by-step explanation:

Quantitative research refers to the collection and the analysis of numerical data. Quantitative research can be used in making predictions, and testing casual relationships.

Quantitative research methods emphasize the numerical analysis of data which can be collected through questionnaires, polls, surveys etc.

Sven determined that the x-coordinate is approximately 3.6 because the point is closer to 4 than 3 and seems to be a little more than halfway between them. What is the approximate value for the y-coordinate? y Almost-equals –1.1 y Almost-equals –1.4 y Almost-equals –1.8 y Almost-equals –1.9

Answers

Answer:

The answer is "[tex]y\approx 1.4[/tex]".

Step-by-step explanation:

In the given question the y-coordinates range between -1 to -2. Its distance between -1 and -2 is near, and less than halfway.

Answer:

b

Step-by-step explanation:

What is the order of rotational symmetry of a regular pentagon?

Answers

Answer:

yeet

Step-by-step explanation:

urmom

Sorry just needed points

use the graph to find the value of y=sin q for the value of q 330​

Answers

Answer:  A) -0.5

Explanation:

Each tickmark is an increment of 30. Start at 270 and move 1 tickmark over to get to 270+30 = 300. Then another tickmark to the right gets us to 330.

Then from here, head straight down until you get to the curve. You should arrive at the point (330, -0.5)

So q = 330 pairs up with y = -0.5

In a 45-45-90 right triangle, what is the ratio of the length of one leg to the length of the other leg?

Answers

Answer:

D) 1:1

Step-by-step explanation:

Answer:
D). 1:1

Explanation:
It would be 90° to 90° which simplifies to 1:1

I hope this helped :)

Cual es!!! Es examen ayudaaaaa

Answers

hello babyyy hpi how you

Please need a help !!
Need explanation
Urgent​

Answers

I think its this, havent checked though
Other Questions
NEED THIS ASAP I AM TIMED WILL MARK BRAINLIST WILL GIVE 10 POINTSRead the excerpt from "Children of the Drug Wars.By sending these children away, "you are handing them a death sentence, says Jos Arnulfo Ochoa Ochoa, an expert in Honduras with World Vision International, a Christian humanitarian aid group. This abrogates international conventions we have signed and undermines our credibility as a humane country. It would be a disgrace if this wealthy nation turned its back on the 52,000 children who have arrived since October, many of them legitimate refugees. The underlined words and phrases have strong negative connotations. How do they support the authors purpose?A. They draw attention to the opinion that the United States is not doing enough to help these children.B. They show that there is a desperate need for all countries to be willing to help refugees.C. They show that the United States has participated in international conventions on how to handle the refugee crisis.D. They make it clear that the United States will lose the respect of other nations if it does not help these children. what is gompertz function Which is the most likely risk in using repetition? A. The reader may feel disoriented. B. The reader may crave a sense of order. C. The reader may fail to see the image. O D. The reader may become bored. MC Qu. 116 CWN Company uses a job order costing... CWN Company uses a job order costing system and last period incurred $70,000 of actual overhead and $100,000 of direct labor. CWN estimates that its overhead next period will be $85,000. It also expects to incur $100,000 of direct labor. If CWN bases applied overhead on direct labor cost, its predetermined overhead rate for the next period should be: Nikola thinks that the model that reflects the growth of smartphones shipped from manufacturers to stores around the world may be logistic rather than exponential. Do you agree with Nikola Which statement best completes the diagram? The fish population of Lake Collins is decreasing at a rate of 5% per year. In 2000 there were about1,150 fish. Write an exponential decay function to model this situation. Then find the population in2006. Tommy's birthday party costs $2.94 for every guest the invites. If there are 4 guests, how much will Tommy's birthday party cost? A wheel has a diameter of 10m and weight 360N what minimum horizontal force is necessary to pull the wheel over a brick 0.1m when a force is applied at the wheel A group of 80 frogs was observed. The mean distance of their hops is 69 inches with a standard deviation of 3.5 inches. How many frogs would you expect to jump more than 72.5 inches? Yall pls help me out Determine the dimensions for T if T = M V^2 A / L^3 where M is a mass, V is a velocity, A is an area, and L is a length. L / TNo dimensionsM L / T^2MM / (L T^2) How many molecular of H2O and O2 are present in 8.5g of H2O2 ? El cuento social poltico y cultural Monique made several batches of soup.Each batch required 3/4 of a pound of potatoes. She used a total of 6 1/2 pounds. How many batches did she make? Should the firms' overseas operations be judged by the standards (legal, economic, cultural, and moral) of the country in which it is operating or should they be judged by the standards of the U.S. market? Someone please help me ASAP!!! Worth 11 points a student estimates the length of a room to be 20 feet. The actual length is 20.25 feet. What is the percent error? I only need the odd numbers answered Assuming equal concentrations and complete dissociation, rank these aqueous solutions by their freezing points from highest to lowest. CoCl3, NH4Cl, Li2SO4 What is the surface area